1. Question: Effective long-term treatment of systolic heart failure with reduced ejection fraction should include which of the following?
2. Question: A 45-year-old man presents with chest pain radiating to the left arm, diaphoresis, and shortness of breath. His ECG shows ST-segment elevation. What is the initial management response?
3. Question: An 80-year-old man with a history of atrial fibrillation presents with sudden-onset unilateral leg pain and pallor. What is the most likely diagnosis?
4. Question: An older adult female presents for her annual examination. She has been on antihypertensive medications for over 20 years, with good control. Laboratory values are within normal ranges. The nurse practitioner is concerned about the patient’s cardiac health risks, due her to weight and her waist circumference. According to the AHA guidelines, which of the following goals is expected for this patient?
exam continue upto 150 questions with solutions
147. Question: Which of the following reflects a reversible cause of pulmonary disease?
148. Question: Your patient has recently traveled to the developing world for a medical service trip and has returned with symptoms of night sweats, weight loss, and hemoptysis. Which of the following interventions represent your priority in their care?
149. Question: Which of the following scenarios suggest a high risk of metabolic acidosis?
150. Question: Which two lobes of the lung are most associated with aspiration pneumonia?
Instituition / Term | |
Term | |
Institution | Chamberlain |
Contributor | Nicole Adams |